Cleaner solution for S&J 8, prob. 2.61.
[course.git] / latex / problems / Serway_and_Jewett_8 / problem03.36.tex
1 \begin{problem*}{3.36}
2 Three displacement vectors of a croquet ball are shown in Figure
3 P3.36, where $|\vect{A}| = 20.0\U{units}$, $|\vect{B}| =
4 40.0\U{units}$, and $|\vect{C}| = 30.0\U{units}$.  Find \Part{a} the
5 resultant in unit-vector notation and \Part{b} the magnitude and
6 direction of the resultant displacement.
7 \begin{center}
8 \begin{asy}
9 import graph;
10 import Mechanics;
11
12 real u = 0.06cm;
13
14 pair a = 20u * dir(90);
15 pair b = 40u * dir(45);
16 pair c = 30u * dir(-45);
17
18 Vector A = Vector((0,0), mag=length(a), dir=degrees(a), "$\vect{A}$");
19 A.draw();
20 Vector B = Vector((0,0), mag=length(b), dir=degrees(b), "$\vect{B}$");
21 B.draw();
22 Vector C = Vector((0,0), mag=length(c), dir=degrees(c), "$\vect{C}$");
23 C.draw();
24
25 xaxis("$x$");
26 yaxis("$y$");
27 \end{asy}
28 \end{center}
29 \end{problem*}
30
31 \begin{solution}
32 \Part{a}
33 \begin{center}
34 \begin{asy}
35 import graph;
36 import Mechanics;
37
38 real u = 0.06cm;
39
40 pair a = 20u * dir(90);
41 pair b = 40u * dir(45);
42 pair c = 30u * dir(-45);
43 pair r = a + b + c;
44
45 Vector A = Vector((0,0), mag=length(a), dir=degrees(a), "$\vect{A}$");
46 A.draw();
47 Vector B = Vector(a, mag=length(b), dir=degrees(b), "$\vect{B}$");
48 B.draw();
49 Vector C = Vector(a+b, mag=length(c), dir=degrees(c), "$\vect{C}$");
50 C.draw();
51 Vector R = Vector((0,0), mag=length(r), dir=degrees(r), "$\vect{r}$");
52 R.draw(labelOffset=-r/2);
53
54 xaxis("$x$");
55 yaxis("$y$");
56 \end{asy}
57 \end{center}
58 The resultant displacement is
59 \begin{align}
60   \vect{r} &= \vect{A} + \vect{B} + \vect{C} \\
61     &= \{  20.0\jhat
62         + 40.0[\cos(45\dg)\ihat + \sin(45\dg)\jhat]
63         + 30.0[\cos(-45\dg)\ihat + \sin(-45\dg)\jhat]\}\U{units} \\
64     &= (49.5\ihat + 27.1\jhat)\U{units}
65 \end{align}
66
67 \Part{b}
68 \begin{align}
69   |\vect{r}| &= \sqrt{\vect{r}_x^2 + \vect{r}_y^2}
70     = \sqrt{49.5^2 + 27.1^2}\U{units}
71     = \ans{56.4\U{units}} \\
72   \theta_\vect{r} &= \arctan\p({\frac{27.1}{49.5}}) = \ans{28.7\dg}
73 \end{align}
74 \end{solution}